Is every commutative ring a limit of noetherian rings?
$begingroup$
Let $mathsf{Noeth}$ be the category of noetherian rings, viewed as a full subcategory of the category $mathsf{CRing}$ of commutative rings with one.
Let $A$ be in $mathsf{CRing}$.
Question 1. Is there a functor from a small category to $mathsf{Noeth}$ whose limit in $mathsf{CRing}$ is $A$?
(I know that there is a functor from a small category to $mathsf{Noeth}$ whose colimit is $A$.)
Let $f:Ato B$ be a morphism in $mathsf{CRing}$ such that the map
$$
circ f:text{Hom}_{mathsf{CRing}}(B,C)totext{Hom}_{mathsf{CRing}}(A,C)
$$
sending $g$ to $gcirc f$ is bijective for all $C$ in $mathsf{Noeth}$.
Question 2. Does this imply that $f$ is an isomorphism?
Yes to Question 1 would imply yes to Question 2.
Question 3. Does the inclusion functor $iota:mathsf{Noeth}tomathsf{CRing}$ commute with colimits? That is, if $Ainmathsf{Noeth}$ is the colimit of a functor $alpha$ from a small category to $mathsf{Noeth}$, is $A$ naturally isomorphic to the colimit of $iotacircalpha$?
Yes to Question 2 would imply yes to Question 3, and yes to Question 3 would imply that many colimits, and in particular many binary coproducts, do not exist in $mathsf{Noeth}$: see this answer of Martin Brandenburg.
One may try to attack the first question as follows:
Let $A$ be in $mathsf{CRing}$ and $I$ the set of those ideals $mathfrak a$ of $A$ such that $A/mathfrak a$ is noetherian. Then $I$ is an ordered set, and thus can be viewed as a category. We can form the limit of the $A/mathfrak a$ with $mathfrak ain I$, and we have a natural morphism from $A$ to this limit. I'd be interested in knowing if this morphism is bijective.
commutative-algebra category-theory noetherian limits-colimits
$endgroup$
|
show 3 more comments
$begingroup$
Let $mathsf{Noeth}$ be the category of noetherian rings, viewed as a full subcategory of the category $mathsf{CRing}$ of commutative rings with one.
Let $A$ be in $mathsf{CRing}$.
Question 1. Is there a functor from a small category to $mathsf{Noeth}$ whose limit in $mathsf{CRing}$ is $A$?
(I know that there is a functor from a small category to $mathsf{Noeth}$ whose colimit is $A$.)
Let $f:Ato B$ be a morphism in $mathsf{CRing}$ such that the map
$$
circ f:text{Hom}_{mathsf{CRing}}(B,C)totext{Hom}_{mathsf{CRing}}(A,C)
$$
sending $g$ to $gcirc f$ is bijective for all $C$ in $mathsf{Noeth}$.
Question 2. Does this imply that $f$ is an isomorphism?
Yes to Question 1 would imply yes to Question 2.
Question 3. Does the inclusion functor $iota:mathsf{Noeth}tomathsf{CRing}$ commute with colimits? That is, if $Ainmathsf{Noeth}$ is the colimit of a functor $alpha$ from a small category to $mathsf{Noeth}$, is $A$ naturally isomorphic to the colimit of $iotacircalpha$?
Yes to Question 2 would imply yes to Question 3, and yes to Question 3 would imply that many colimits, and in particular many binary coproducts, do not exist in $mathsf{Noeth}$: see this answer of Martin Brandenburg.
One may try to attack the first question as follows:
Let $A$ be in $mathsf{CRing}$ and $I$ the set of those ideals $mathfrak a$ of $A$ such that $A/mathfrak a$ is noetherian. Then $I$ is an ordered set, and thus can be viewed as a category. We can form the limit of the $A/mathfrak a$ with $mathfrak ain I$, and we have a natural morphism from $A$ to this limit. I'd be interested in knowing if this morphism is bijective.
commutative-algebra category-theory noetherian limits-colimits
$endgroup$
$begingroup$
I guess by question 1 you mean if there's a functor $F$ from a small category to ${bf Noeth}$ such that $A$ is the limit of $EF$. ($E:{bf Noeth} rightarrow {bf CRing}$ is the inclusion functor.)
$endgroup$
– sqtrat
Feb 1 at 13:24
$begingroup$
@sqtrat - Thanks! Yes. I've added "in $mathsf{CRing}$". I hope it's clear enough now.
$endgroup$
– Pierre-Yves Gaillard
Feb 1 at 13:41
$begingroup$
Doesn't question 2 follow from the fact that a ring is colimit of noetherian rings ?(plug in $C_i$, where $varinjlim C_i =A$ to get $id_A$ has an antecedent, then plug in $D_i$ where $varinjlim D_i = B$ to get that the inverse we get was a $2$-sided inverse) (I used $varinjlim$ to denote the colimit, because in fact we can choose the colimit to be a "direct limit")
$endgroup$
– Max
Feb 1 at 14:12
1
$begingroup$
@Pierre-YvesGaillard : sorry I actually wrote it down and it didn't work
$endgroup$
– Max
Feb 1 at 16:53
1
$begingroup$
Crossposted on MathOverflow: mathoverflow.net/q/323136/461
$endgroup$
– Pierre-Yves Gaillard
Feb 13 at 12:52
|
show 3 more comments
$begingroup$
Let $mathsf{Noeth}$ be the category of noetherian rings, viewed as a full subcategory of the category $mathsf{CRing}$ of commutative rings with one.
Let $A$ be in $mathsf{CRing}$.
Question 1. Is there a functor from a small category to $mathsf{Noeth}$ whose limit in $mathsf{CRing}$ is $A$?
(I know that there is a functor from a small category to $mathsf{Noeth}$ whose colimit is $A$.)
Let $f:Ato B$ be a morphism in $mathsf{CRing}$ such that the map
$$
circ f:text{Hom}_{mathsf{CRing}}(B,C)totext{Hom}_{mathsf{CRing}}(A,C)
$$
sending $g$ to $gcirc f$ is bijective for all $C$ in $mathsf{Noeth}$.
Question 2. Does this imply that $f$ is an isomorphism?
Yes to Question 1 would imply yes to Question 2.
Question 3. Does the inclusion functor $iota:mathsf{Noeth}tomathsf{CRing}$ commute with colimits? That is, if $Ainmathsf{Noeth}$ is the colimit of a functor $alpha$ from a small category to $mathsf{Noeth}$, is $A$ naturally isomorphic to the colimit of $iotacircalpha$?
Yes to Question 2 would imply yes to Question 3, and yes to Question 3 would imply that many colimits, and in particular many binary coproducts, do not exist in $mathsf{Noeth}$: see this answer of Martin Brandenburg.
One may try to attack the first question as follows:
Let $A$ be in $mathsf{CRing}$ and $I$ the set of those ideals $mathfrak a$ of $A$ such that $A/mathfrak a$ is noetherian. Then $I$ is an ordered set, and thus can be viewed as a category. We can form the limit of the $A/mathfrak a$ with $mathfrak ain I$, and we have a natural morphism from $A$ to this limit. I'd be interested in knowing if this morphism is bijective.
commutative-algebra category-theory noetherian limits-colimits
$endgroup$
Let $mathsf{Noeth}$ be the category of noetherian rings, viewed as a full subcategory of the category $mathsf{CRing}$ of commutative rings with one.
Let $A$ be in $mathsf{CRing}$.
Question 1. Is there a functor from a small category to $mathsf{Noeth}$ whose limit in $mathsf{CRing}$ is $A$?
(I know that there is a functor from a small category to $mathsf{Noeth}$ whose colimit is $A$.)
Let $f:Ato B$ be a morphism in $mathsf{CRing}$ such that the map
$$
circ f:text{Hom}_{mathsf{CRing}}(B,C)totext{Hom}_{mathsf{CRing}}(A,C)
$$
sending $g$ to $gcirc f$ is bijective for all $C$ in $mathsf{Noeth}$.
Question 2. Does this imply that $f$ is an isomorphism?
Yes to Question 1 would imply yes to Question 2.
Question 3. Does the inclusion functor $iota:mathsf{Noeth}tomathsf{CRing}$ commute with colimits? That is, if $Ainmathsf{Noeth}$ is the colimit of a functor $alpha$ from a small category to $mathsf{Noeth}$, is $A$ naturally isomorphic to the colimit of $iotacircalpha$?
Yes to Question 2 would imply yes to Question 3, and yes to Question 3 would imply that many colimits, and in particular many binary coproducts, do not exist in $mathsf{Noeth}$: see this answer of Martin Brandenburg.
One may try to attack the first question as follows:
Let $A$ be in $mathsf{CRing}$ and $I$ the set of those ideals $mathfrak a$ of $A$ such that $A/mathfrak a$ is noetherian. Then $I$ is an ordered set, and thus can be viewed as a category. We can form the limit of the $A/mathfrak a$ with $mathfrak ain I$, and we have a natural morphism from $A$ to this limit. I'd be interested in knowing if this morphism is bijective.
commutative-algebra category-theory noetherian limits-colimits
commutative-algebra category-theory noetherian limits-colimits
edited Feb 1 at 13:39
Pierre-Yves Gaillard
asked Feb 1 at 12:11
Pierre-Yves GaillardPierre-Yves Gaillard
13.5k23184
13.5k23184
$begingroup$
I guess by question 1 you mean if there's a functor $F$ from a small category to ${bf Noeth}$ such that $A$ is the limit of $EF$. ($E:{bf Noeth} rightarrow {bf CRing}$ is the inclusion functor.)
$endgroup$
– sqtrat
Feb 1 at 13:24
$begingroup$
@sqtrat - Thanks! Yes. I've added "in $mathsf{CRing}$". I hope it's clear enough now.
$endgroup$
– Pierre-Yves Gaillard
Feb 1 at 13:41
$begingroup$
Doesn't question 2 follow from the fact that a ring is colimit of noetherian rings ?(plug in $C_i$, where $varinjlim C_i =A$ to get $id_A$ has an antecedent, then plug in $D_i$ where $varinjlim D_i = B$ to get that the inverse we get was a $2$-sided inverse) (I used $varinjlim$ to denote the colimit, because in fact we can choose the colimit to be a "direct limit")
$endgroup$
– Max
Feb 1 at 14:12
1
$begingroup$
@Pierre-YvesGaillard : sorry I actually wrote it down and it didn't work
$endgroup$
– Max
Feb 1 at 16:53
1
$begingroup$
Crossposted on MathOverflow: mathoverflow.net/q/323136/461
$endgroup$
– Pierre-Yves Gaillard
Feb 13 at 12:52
|
show 3 more comments
$begingroup$
I guess by question 1 you mean if there's a functor $F$ from a small category to ${bf Noeth}$ such that $A$ is the limit of $EF$. ($E:{bf Noeth} rightarrow {bf CRing}$ is the inclusion functor.)
$endgroup$
– sqtrat
Feb 1 at 13:24
$begingroup$
@sqtrat - Thanks! Yes. I've added "in $mathsf{CRing}$". I hope it's clear enough now.
$endgroup$
– Pierre-Yves Gaillard
Feb 1 at 13:41
$begingroup$
Doesn't question 2 follow from the fact that a ring is colimit of noetherian rings ?(plug in $C_i$, where $varinjlim C_i =A$ to get $id_A$ has an antecedent, then plug in $D_i$ where $varinjlim D_i = B$ to get that the inverse we get was a $2$-sided inverse) (I used $varinjlim$ to denote the colimit, because in fact we can choose the colimit to be a "direct limit")
$endgroup$
– Max
Feb 1 at 14:12
1
$begingroup$
@Pierre-YvesGaillard : sorry I actually wrote it down and it didn't work
$endgroup$
– Max
Feb 1 at 16:53
1
$begingroup$
Crossposted on MathOverflow: mathoverflow.net/q/323136/461
$endgroup$
– Pierre-Yves Gaillard
Feb 13 at 12:52
$begingroup$
I guess by question 1 you mean if there's a functor $F$ from a small category to ${bf Noeth}$ such that $A$ is the limit of $EF$. ($E:{bf Noeth} rightarrow {bf CRing}$ is the inclusion functor.)
$endgroup$
– sqtrat
Feb 1 at 13:24
$begingroup$
I guess by question 1 you mean if there's a functor $F$ from a small category to ${bf Noeth}$ such that $A$ is the limit of $EF$. ($E:{bf Noeth} rightarrow {bf CRing}$ is the inclusion functor.)
$endgroup$
– sqtrat
Feb 1 at 13:24
$begingroup$
@sqtrat - Thanks! Yes. I've added "in $mathsf{CRing}$". I hope it's clear enough now.
$endgroup$
– Pierre-Yves Gaillard
Feb 1 at 13:41
$begingroup$
@sqtrat - Thanks! Yes. I've added "in $mathsf{CRing}$". I hope it's clear enough now.
$endgroup$
– Pierre-Yves Gaillard
Feb 1 at 13:41
$begingroup$
Doesn't question 2 follow from the fact that a ring is colimit of noetherian rings ?(plug in $C_i$, where $varinjlim C_i =A$ to get $id_A$ has an antecedent, then plug in $D_i$ where $varinjlim D_i = B$ to get that the inverse we get was a $2$-sided inverse) (I used $varinjlim$ to denote the colimit, because in fact we can choose the colimit to be a "direct limit")
$endgroup$
– Max
Feb 1 at 14:12
$begingroup$
Doesn't question 2 follow from the fact that a ring is colimit of noetherian rings ?(plug in $C_i$, where $varinjlim C_i =A$ to get $id_A$ has an antecedent, then plug in $D_i$ where $varinjlim D_i = B$ to get that the inverse we get was a $2$-sided inverse) (I used $varinjlim$ to denote the colimit, because in fact we can choose the colimit to be a "direct limit")
$endgroup$
– Max
Feb 1 at 14:12
1
1
$begingroup$
@Pierre-YvesGaillard : sorry I actually wrote it down and it didn't work
$endgroup$
– Max
Feb 1 at 16:53
$begingroup$
@Pierre-YvesGaillard : sorry I actually wrote it down and it didn't work
$endgroup$
– Max
Feb 1 at 16:53
1
1
$begingroup$
Crossposted on MathOverflow: mathoverflow.net/q/323136/461
$endgroup$
– Pierre-Yves Gaillard
Feb 13 at 12:52
$begingroup$
Crossposted on MathOverflow: mathoverflow.net/q/323136/461
$endgroup$
– Pierre-Yves Gaillard
Feb 13 at 12:52
|
show 3 more comments
0
active
oldest
votes
Your Answer
StackExchange.ifUsing("editor", function () {
return StackExchange.using("mathjaxEditing", function () {
StackExchange.MarkdownEditor.creationCallbacks.add(function (editor, postfix) {
StackExchange.mathjaxEditing.prepareWmdForMathJax(editor, postfix, [["$", "$"], ["\\(","\\)"]]);
});
});
}, "mathjax-editing");
StackExchange.ready(function() {
var channelOptions = {
tags: "".split(" "),
id: "69"
};
initTagRenderer("".split(" "), "".split(" "), channelOptions);
StackExchange.using("externalEditor", function() {
// Have to fire editor after snippets, if snippets enabled
if (StackExchange.settings.snippets.snippetsEnabled) {
StackExchange.using("snippets", function() {
createEditor();
});
}
else {
createEditor();
}
});
function createEditor() {
StackExchange.prepareEditor({
heartbeatType: 'answer',
autoActivateHeartbeat: false,
convertImagesToLinks: true,
noModals: true,
showLowRepImageUploadWarning: true,
reputationToPostImages: 10,
bindNavPrevention: true,
postfix: "",
imageUploader: {
brandingHtml: "Powered by u003ca class="icon-imgur-white" href="https://imgur.com/"u003eu003c/au003e",
contentPolicyHtml: "User contributions licensed under u003ca href="https://creativecommons.org/licenses/by-sa/3.0/"u003ecc by-sa 3.0 with attribution requiredu003c/au003e u003ca href="https://stackoverflow.com/legal/content-policy"u003e(content policy)u003c/au003e",
allowUrls: true
},
noCode: true, onDemand: true,
discardSelector: ".discard-answer"
,immediatelyShowMarkdownHelp:true
});
}
});
Sign up or log in
StackExchange.ready(function () {
StackExchange.helpers.onClickDraftSave('#login-link');
});
Sign up using Google
Sign up using Facebook
Sign up using Email and Password
Post as a guest
Required, but never shown
StackExchange.ready(
function () {
StackExchange.openid.initPostLogin('.new-post-login', 'https%3a%2f%2fmath.stackexchange.com%2fquestions%2f3096154%2fis-every-commutative-ring-a-limit-of-noetherian-rings%23new-answer', 'question_page');
}
);
Post as a guest
Required, but never shown
0
active
oldest
votes
0
active
oldest
votes
active
oldest
votes
active
oldest
votes
Thanks for contributing an answer to Mathematics Stack Exchange!
- Please be sure to answer the question. Provide details and share your research!
But avoid …
- Asking for help, clarification, or responding to other answers.
- Making statements based on opinion; back them up with references or personal experience.
Use MathJax to format equations. MathJax reference.
To learn more, see our tips on writing great answers.
Sign up or log in
StackExchange.ready(function () {
StackExchange.helpers.onClickDraftSave('#login-link');
});
Sign up using Google
Sign up using Facebook
Sign up using Email and Password
Post as a guest
Required, but never shown
StackExchange.ready(
function () {
StackExchange.openid.initPostLogin('.new-post-login', 'https%3a%2f%2fmath.stackexchange.com%2fquestions%2f3096154%2fis-every-commutative-ring-a-limit-of-noetherian-rings%23new-answer', 'question_page');
}
);
Post as a guest
Required, but never shown
Sign up or log in
StackExchange.ready(function () {
StackExchange.helpers.onClickDraftSave('#login-link');
});
Sign up using Google
Sign up using Facebook
Sign up using Email and Password
Post as a guest
Required, but never shown
Sign up or log in
StackExchange.ready(function () {
StackExchange.helpers.onClickDraftSave('#login-link');
});
Sign up using Google
Sign up using Facebook
Sign up using Email and Password
Post as a guest
Required, but never shown
Sign up or log in
StackExchange.ready(function () {
StackExchange.helpers.onClickDraftSave('#login-link');
});
Sign up using Google
Sign up using Facebook
Sign up using Email and Password
Sign up using Google
Sign up using Facebook
Sign up using Email and Password
Post as a guest
Required, but never shown
Required, but never shown
Required, but never shown
Required, but never shown
Required, but never shown
Required, but never shown
Required, but never shown
Required, but never shown
Required, but never shown
$begingroup$
I guess by question 1 you mean if there's a functor $F$ from a small category to ${bf Noeth}$ such that $A$ is the limit of $EF$. ($E:{bf Noeth} rightarrow {bf CRing}$ is the inclusion functor.)
$endgroup$
– sqtrat
Feb 1 at 13:24
$begingroup$
@sqtrat - Thanks! Yes. I've added "in $mathsf{CRing}$". I hope it's clear enough now.
$endgroup$
– Pierre-Yves Gaillard
Feb 1 at 13:41
$begingroup$
Doesn't question 2 follow from the fact that a ring is colimit of noetherian rings ?(plug in $C_i$, where $varinjlim C_i =A$ to get $id_A$ has an antecedent, then plug in $D_i$ where $varinjlim D_i = B$ to get that the inverse we get was a $2$-sided inverse) (I used $varinjlim$ to denote the colimit, because in fact we can choose the colimit to be a "direct limit")
$endgroup$
– Max
Feb 1 at 14:12
1
$begingroup$
@Pierre-YvesGaillard : sorry I actually wrote it down and it didn't work
$endgroup$
– Max
Feb 1 at 16:53
1
$begingroup$
Crossposted on MathOverflow: mathoverflow.net/q/323136/461
$endgroup$
– Pierre-Yves Gaillard
Feb 13 at 12:52